Ask Question
18 April, 00:57

If mean is 6 wat number could be p given se t of numbers r 4,5,8, p, 7,6

+2
Answers (1)
  1. 18 April, 03:01
    0
    Let's solve your equation step-by-step. (4+5 + 8 + p + 7 + 6) / 6 = 6 Step 1: Multiply both sides by 6. (4+5 + 8 + p + 7 + 6) / 6 = 6 (4+5 + 8 + p + 7 + 6) / (6) * (6) = (6) * (6) 4+5 + 8 + p + 7 + 6 = 36 Step 2: Simplify both sides of the equation. p+30 = 36 Step 3: Subtract 30 from both sides. p+30 - 30 = 36-30 p=6 Answer: p = 6
Know the Answer?
Not Sure About the Answer?
Get an answer to your question ✅ “If mean is 6 wat number could be p given se t of numbers r 4,5,8, p, 7,6 ...” in 📙 Mathematics if there is no answer or all answers are wrong, use a search bar and try to find the answer among similar questions.
Search for Other Answers